You are on page 1of 39

Section 5.

3 – Properties of
Continuous Functions

The goal of this section is to


discuss important properties of
continuous functions.

© Markus Pomper, 2020


Bounded Functions
  Suppose that is a function.

 We say that is bounded

  if is a bounded subset of .

  That is, there exists a real number ,

  such that for all .


Bounded Functions
  Suppose now that is a continuous function.

  If is bounded, does it follow that is bounded?

The answer is NO.


3
  Suppose that .
2

  We let .
1
  Then is continuous.

  But , which is unbounded. 1


Compactness
  The previous example shows that the image

  of a continuous function need not be bounded, even if is bounded.

  Recall that a subset of is compact

  if and only if is closed and bounded.

(Heine-Borel Theorem 3.5.5)

We will show that the continuous image of a compact set is compact.


Theorem 5.3.2
  Let be a compact subset of

  and suppose that is continuous.

  Then is compact.
Proof
  We must show that is compact.

  To do this, we must prove that every open cover of

can be reduced to a finite subcover.

  Suppose now that is an open cover of .

  Because is continuous, and each is an open subset of ,

we can apply Theorem 5.2.14:

  There exist open sets of , such that


Proof
  We first observe that

  implies that .

  Further, is and open cover of , i.e.,


 
𝑓 ( 𝐷 ) ⊆ {𝐺 𝛼 }
Therefore,
  −1
𝐷⊆ 𝑓 ( {𝐺 𝛼 })
and then

𝐷⊆ { 𝑓 ( 𝐺𝛼 ) } ⊆ { 𝐻 𝛼 }
  −1  
Proof
  Therefore, is an open cover of .

  By hypothesis, is compact.

  Therefore, there exists a finite subcover of ,

  say

  But then

and therefore

𝑓  ( 𝐷 ) ⊆ 𝑓 ( 𝐻 𝛼 ∩ 𝐷 ) ∪ 𝑓 ( 𝐻 𝛼 ∩ 𝐷) ∪ … ∪𝑓 ( 𝐻 𝛼 ∩ 𝐷 )
1 2 𝑛

⊆𝐺
  𝛼 ∪𝐺 𝛼 ∪… ∪𝐺 𝛼
1 2 𝑛
Proof
  We proved that

  Therefore, the open cover of ,

  has a finite subcover .

  Therefore, is compact.
Corollary 5.3.3
  Let be a compact set

  and let be a continuous function

  Then assumes a maximum and a minimum on .

  That is, there exist and in , such that

  for all .

  Thus, is the minimum value of the function

  and is the maximum value of the function


Proof
y2
  By hypothesis, is compact.

  and is a continuous function.


𝑓  ( 𝐷 )
 𝑦=𝑓 ( 𝑥)
  By Theorem 5.3.2, is compact. y1

We proved in Lemma 3.5.4 D

that compact sets have a maximum and a minimum element.

  We therefore let   and

  That is for all .


Proof
y2
  Because ,
  there exists , such that .
𝑓  ( 𝐷 )
  Because ,  𝑦=𝑓 ( 𝑥)
  there exists , such that .
y1
  Because every
D
  is of the form for some , x1 x2
We can re-write the statement
  for all as follows:

  for all .

This proves the statement of the Corollary.


Now Try This!
Practice 5.3.4

Exercise 5.3.3

STOP
Is Compactness Needed?
The statement of the Corollary is false

when the requirement of compactness is removed.

Recall that compact subsets of the real numbers are closed and bounded.

  The set is closed, but not bounded.

  The function ,

  given by is continuous.

  But ,

  so the function does not assume a maximum on .


Is Compactness Needed?
  Give an example of a function ,

  where is bounded, but not closed,

  and does not have a maximum.

STOP
Compact Intervals
  We have seen that compact subsets of are closed and bounded.

We now consider compact intervals.

  That is, intervals of the form .

  Continuous functions have additional useful properties.


Lemma 5.3.5
  Let be a continuous function

  and suppose that .  𝑦=𝑓 ( 𝑥 )

𝑓  ( 𝑏 )
  Then there exists a ,

  such that .
a c b

𝑓  ( 𝑎 )
Proof
  To prove the Lemma, we consider the collection of all , for which .

  We let .
 𝑦=𝑓 ( 𝑥 )

  By hypothesis, , so , 𝑓  ( 𝑏 )

  and therefore .
S
  is also bounded above by . a b
c
 Therefore, has a supremum. 𝑓  ( 𝑎 )
  We let .
Proof
  We now claim that .

  We will prove that and are both impossible.


 𝑦=𝑓 ( 𝑥 )
  We first show that is impossible.
f (b)

  Suppose (aiming for a contradiction) that .


S
  Then there exists a neighborhood of , such that a b
c

  for all . f (a)

This was proved in Exercise 5.2.13 (See details?)


Proof
  Because , we let .

  Then .

  Because is continuous at , there exists ,

  such that for all and .

We may re-write the first inequality as

  .

  Only considering the right inequality, and noting that gives us

  for all .
Proof
  We first observe that .  𝑦=𝑓 ( 𝑥 )

  This is true because . 𝑓  ( 𝑏 )


  Therefore, there exists a
S U
  such that .
a c b
  Therefore ,
𝑓  ( 𝑎 )
  and by definition, .

  Because , we obtain that is not an upper bound for .

  This is a contradiction to being the supremum of .

  Therefore, is not possible.


Proof
  We now consider the possibility that .
  If ,
  there exists a neighborhood of ,  𝑦=𝑓 ( 𝑥 )

  such that for all . 𝑓  ( 𝑏 )


  Because by hypothesis,   and ,
S U
  there exists a ,  such that .
  We observe that for all . a pc b

  Therefore, no points of are in . 𝑓  ( 𝑎 )


  for all .
  Therefore, is an upper bound for .
  This contradict that is the least upper bound of .
Proof
  We showed that is impossible

  and that is also impossible.

  The only other possibility is that .

This proves the Lemma.


Now Try This!
Exercise 5.3.6

STOP
Intermediate Value Theorem
The Lemma can be restated as follows:

  If is a continuous function,

  and if and have opposing signs,

  then has a zero in the interval

The next Theorem generalizes this result.

  It states that a continuous function

  assumes all values between and .


Theorem 5.3.6
  Suppose that is a continuous function.

  and that is any number between and .  𝑦=𝑓 ( 𝑥 )

  (This means: If , then 𝑓  ( 𝑏 )

  and if then ).
k
  Then there exists a ,

  such that . 𝑓  (𝑎)

a c b
Proof
We consider two cases:

  Case 1: .

  In this case we consider the function ,

  given by .

  Because , we obtain

  ,

  and therefore .
Proof
We can therefore apply Lemma 5.3.5 and obtain

  that there exists a ,

  such that .

  By definition of , this means , or .

  This proves the Theorem for the case where .


Proof
  Case 2: Suppose now that .

  In this case, we define the function

  by setting .

Finish the proof of the Theorem.

STOP
𝑓  ( 𝑥 )=𝑥 𝑛
Example 5.3.7 f (b)

  Let be a positive number, and let .

  We now show that as an -th root.

  That is, we show that there exists a , such that .

  We let .
k
  Let .

  We showed that is continuous.

  Further, .

  . c k 𝑘  +1
  Because ,

  there exists , such that .


Now Try This!
Exercise 5.3.4

Exercise 5.3.5

Exercise 5.3.8

STOP
Fixed Points
  We now consider a continuous function .

 We claim that the function b

has a fixed point,


c
  that is, there exists a point ,

  such that . a

  That is, the graph intersects the line .


a c b
Fixed Points
  We consider the function ,

  defined by .

  Because the function is ,

  we have for all

  Therefore, and .

  If either or , we have a fixed point.

  Otherwise and .

  or and .
Now Try This!
Exercise 5.3.7

STOP
Theorem 5.3.10
  Let be a compact interval and suppose that is a continuous function.

  Then the set is a compact interval.


Proof
  Corollary 5.3.3 implies that there exist and in such that

  for all .

  Let and .

  Then .

  If , then , and we are done.


Proof
  If and ,

  Then Theorem 5.3.6 implies that for some between and .

  Thus, .

  Finally, since and are in , we have .

  Hence, is the compact interval


Now Try This!
Exercise 5.3.13

STOP
Homework
Exercise 5.3.4

Exercise 5.3.5

Exercise 5.3.7

Exercise 5.3.13

You might also like